Recent questions tagged quantitative-aptitude

1 votes
1 answer
781
2 votes
1 answer
783
Let $X=\frac{1}{1001} + \frac{1}{1002} + \frac{1}{1003} + \cdots + \frac{1}{3001}$. Then,$X \lt1$$X\gt3/2$$1\lt X\lt 3/2$none of the above holds
2 votes
1 answer
784
1 votes
2 answers
785
Let $\cos ^6 \theta = a_6 \cos 6 \theta + a_5 \cos 5 \theta + a_4 \cos 4 \theta + a_3 \cos 3 \theta + a_2 \cos 2 \theta + a_1 \cos \theta +a_0$. Then $a_0$ is$0$$1/32$$...
1 votes
1 answer
786
In a triangle $ABC$, $AD$ is the median. If length of $AB$ is $7$, length of $AC$ is $15$ and length of $BC$ is $10$ then length of $AD$ equals$\sqrt{125}$$69/5$$\sqrt{11...
1 votes
1 answer
787
Let $k$ and $n$ be integers greater than $1$. Then $(kn)!$ is not necessarily divisible by$(n!)^k$$(k!)^n$$n! \cdot k! \cdot$$2^{kn}$
2 votes
4 answers
788
The sequence $\dfrac{1}{\log_3 2}, \: \dfrac{1}{\log_6 2}, \: \dfrac{1}{\log_{12} 2}, \: \dfrac{1}{\log_{24} 2} \dots $ is inArithmetic progression (AP)Geometric progress...
1 votes
4 answers
789
Let $S=\{6, 10, 7, 13, 5, 12, 8, 11, 9\}$ and $a=\underset{x \in S}{\Sigma} (x-9)^2$ & $b = \underset{x \in S}{\Sigma} (x-10)^2$. Then$a <b$$a>b$$a=b$None of these
0 votes
2 answers
790
If $\tan x=p+1$ and $\tan y=p-1$, then the value of $2 \cot (x-y)$ is$2p$$p^2$$(p+1)(p-1)$$\frac{2p}{p^2-1}$
1 votes
1 answer
791
The coefficient of $x^2$ in the product $(1+x)(1+2x)(1+3x) \dots (1+10x)$ is$1320$$1420$$1120$None of these
0 votes
1 answer
792
Let $x^2-2(4k-1)x+15k^2-2k-7>0$ for any real value of $x$. Then the integer value of $k$ is$2$$4$$3$$1$
1 votes
2 answers
793
Let $S=\{0, 1, 2, \cdots 25\}$ and $T=\{n \in S: n^2+3n+2$ is divisible by $6\}$. Then the number of elements in the set $T$ is$16$$17$$18$$10$
2 votes
1 answer
794
0 votes
1 answer
795
The $5000$th term of the sequence $1,2,2, 3,3,3,4,4,4,4, \cdots$ is$98$$99$$100$$101$
1 votes
1 answer
796
0 votes
1 answer
797
For all the natural number $n \geq 3, \: n^2+1$ isdivisible by $3$not divisible by $3$divisible by $9$None of these
0 votes
1 answer
798
For natural numbers $n$, the inequality $2^n >2n+1$ is valid when$n \geq 3$$n < 3$$n=3$None of these
0 votes
1 answer
799
The smallest integer $n$ for which $1+2+2^2+2^3+2^4+ \cdots +2^n$ exceeds $9999$, given that $\log_{10} 2=0.30103$, is$12$$13$$14$None of these
0 votes
2 answers
800
The shaded region in the following diagram represents the relation$y \leq x$$\mid y \mid \leq \mid x \mid$$y \leq \mid x \mid$$\mid y \mid \leq x$
0 votes
1 answer
801
0 votes
1 answer
802
The expression $3^{2n+1} + 2^{n+2}$ is divisible by $7$ forall positive integer values of $n$all non-negative integer values of $n$only even integer values of $n$only odd...
2 votes
2 answers
803
The total number of factors of $3528$ greater than $1$ but less than $3528$ is$35$$36$$34$None of these
1 votes
2 answers
804
The value of $\log _2 e – \log _4 e + \log _8 e – \log _{16} e + \log_{32} e – \cdots$ is$-1$$0$$1$None of these
1 votes
1 answer
805
If $\alpha$ and $\beta$ be the roots of the equation $x^2+3x+4=0$, then the equation with roots $(\alpha + \beta)^2$ and $(\alpha – \beta)^2$ is$x^2+2x+63=0$$x^2-63x+2=...
0 votes
1 answer
806
If $r$ be the ratio of the roots of the equation $ax^{2}+bx+c=0,$ then $\frac{r}{b}=\frac{r+1}{ac}$$\frac{r+1}{b}=\frac{r}{ac}$$\frac{(r+1)^{2}}{r}=\frac{b^{2}}{ac}$$\lef...
3 votes
1 answer
808
The condition that ensures that the roots of the equation $x^3-px^2+qx-r=0$ are in $H.P.$ is$r^2-9pqr+q^3=0$$27r^2-9pqr+3q^3=0$$3r^3-27pqr-9q^3=0$$27r^2-9pqr+2q^3=0$
0 votes
1 answer
810
The length of the chord on the straight line $3x-4y+5=0$ intercepted by the circle passing through the points $(1,2), (3,-4)$ and $(5,6)$ is$12$$14$$16$$18$